Posts

Showing posts from January 2, 2019

Weak LLN problem

Image
0 Suppose $(X_n)$ is a sequence of r.v's satisfying $P(X_n=pmln (n))=frac{1}{2}$ for each $n=1,2dots$ . I am trying to show that $(X_n)$ satisfies the weak LLN. The idea is to show that $P(overline{X_n}>varepsilon)$ tends to 0, but i am unsure how to do so. probability probability-theory stochastic-processes share | cite | improve this question edited Dec 2 '18 at 6:54 GNUSupporter 8964民主女神 地下教會 12.8k 7 24 45 asked Dec 2 '18 at 6:53 user593295